For ωn1=k=1∏ncot2(2n+1kπ)(k=0∑2ncot(x+2n+1kπ))
FindΩ=n→∞lim(x→4n+2πlim(tan(4n+2π)tanx)ωn)
This is the proposal by teachers Marian Ursărescu and Florică Anastase, Romania and published by Romanian Mathematical Magazine.
Initially the answer of proposers agreed with me but later we had a slight different conclusions. As the per the proposers they arrived to πe41.
If the answer of proposers is correct then probably I think I had done mistakes while evaluating the limits. I even tried on wolfram alpha it agrees with me ( doesn't provide the closed form but it approximates to my closed form) . Can anybody help me to correct my mistakes ?
Thank you ! :)
Solution
Here i wish to share my solution.
Firstly we recall Euler's formulaeimx=(cosmx+isinmx)=(cosx+isinx)m and further solving gives sinmxcosmx+isinmx=r=0∑m(rm)cotm−rxir=r=0∑⌈2m⌉+l(−1)r(2rm)cotm−2rx+ir=1∑⌈2m⌉(2r−1m)(−1)rcotm−2r+1x where l=1,0 if m is even and odd respectively. We note that latter sum is the imaginary part and hence equating we get
r=1∑⌈2m⌉(2r−1m)cotm−2r+1x=sinmxsinmx on setting m=2n+1,x=mkπ and further expansion we get (12n+1)cot2n2n+1kπ−(32n+1)cot2n−22n+1kπ+⋯+(2n+12n+1)=0 as the polynomial equation is of even degree and hence by Vieta's formula we have k=1∏ncot2(2n+1kπ)=(−1)2n(12n+1)(2n+12n+1)=2n+11 secondly we use the identity k=0∏n−1sin(x+nkπ)=2n−1sinnx, taking log on both side and on differentiating with respect to x, we get that, ie dxdlog(k=0∏n−1sin(x+nkπ))=dxdlog(2n−1sinnx)⇒k=0∑n−1cot(x+nkπ)=ncot(nx). Replacing n by 2n+1 we yield k=0∑2ncot(x+2n+1kπ)=(2n+1)cot((2n+1)x)and we deduce that ωn=(2n+1(2n+1)cot((2n+1)x))−1=tan((2n+1)x).
Call L(n)=x→4n+2πlim(tanβtanx)ωn. As we can observe that we have 1∞ limit form. Since as soon as x→4n+21,function wn→∞ and tanβtanx→1 with β=4n+2π. So we can either make the direct use the formula for 1∞ or without of it too. ie
x→βlim(tanβtanx)ωn=x→βlim(1+tanβtanx−1)ωn=x→βlim(1+tanβtanx−11)ωn=exp(x→βlim(tanβtanx−1)tan((2n+1)x))=exp(x→βlim(tanβcot((2n+1)x)tanx−tanβ)). Here we have the limit of the form 00 so we use L-hopital's rule to get
exp(x→βlim−(2n+1)csc2((2n+1)x)tanβsec2x)=exp(−sinβ(2n+1)secβ). And therefore,
n→∞limL(n)=exp(−n→∞limtan4n+2ππ(4n+2)2π)=exp(−π2n→∞limtan4n+2π(4n+2)π)=e−π2=πe21
This discussion board is a place to discuss our Daily Challenges and the math and science
related to those challenges. Explanations are more than just a solution — they should
explain the steps and thinking strategies that you used to obtain the solution. Comments
should further the discussion of math and science.
When posting on Brilliant:
Use the emojis to react to an explanation, whether you're congratulating a job well done , or just really confused .
Ask specific questions about the challenge or the steps in somebody's explanation. Well-posed questions can add a lot to the discussion, but posting "I don't understand!" doesn't help anyone.
Try to contribute something new to the discussion, whether it is an extension, generalization or other idea related to the challenge.
Stay on topic — we're all here to learn more about math and science, not to hear about your favorite get-rich-quick scheme or current world events.
Markdown
Appears as
*italics* or _italics_
italics
**bold** or __bold__
bold
- bulleted - list
bulleted
list
1. numbered 2. list
numbered
list
Note: you must add a full line of space before and after lists for them to show up correctly
I tend to agree with you The inner limit is
x→βlim(tanβtanx)tan(2βπx)=y→0lim(tanβtan(β−y))cot(2βπy)=y→0limF(y)
where β=4n+2π, so that
y→0limlnF(y)=y→0limtan2βπyln(tanβtan(β−y))=y→0lim2βπsec22βπy−tanβ−tanysec2y−1+tanβtanytanβsec2y=−π2β(tanβ+cotβ)=−πsin2β4β
which in turn tends to −π2 as n→∞, so that β→0. Exponentiating gives the limit πe21.
I have not looked at the original question. You have transcribed it correctly?
Yes, I have it correctly, Sir. I don't know how to attach the photo of it here in comment section so I'm here providing the link of it Original problem as I got this problem on Facebook wall of proposer. Please do check once. Thank your very much for your response. :)
Easy Math Editor
This discussion board is a place to discuss our Daily Challenges and the math and science related to those challenges. Explanations are more than just a solution — they should explain the steps and thinking strategies that you used to obtain the solution. Comments should further the discussion of math and science.
When posting on Brilliant:
*italics*
or_italics_
**bold**
or__bold__
paragraph 1
paragraph 2
[example link](https://brilliant.org)
> This is a quote
\(
...\)
or\[
...\]
to ensure proper formatting.2 \times 3
2^{34}
a_{i-1}
\frac{2}{3}
\sqrt{2}
\sum_{i=1}^3
\sin \theta
\boxed{123}
Comments
@Mark Hennings Sir, can you help me?
Thank you !! :)
Log in to reply
I tend to agree with you The inner limit is x→βlim(tanβtanx)tan(2βπx)=y→0lim(tanβtan(β−y))cot(2βπy)=y→0limF(y) where β=4n+2π, so that y→0limlnF(y)=y→0limtan2βπyln(tanβtan(β−y))=y→0lim2βπsec22βπy−tanβ−tanysec2y−1+tanβtanytanβsec2y=−π2β(tanβ+cotβ)=−πsin2β4β which in turn tends to −π2 as n→∞, so that β→0. Exponentiating gives the limit πe21.
I have not looked at the original question. You have transcribed it correctly?
Log in to reply
Yes, I have it correctly, Sir. I don't know how to attach the photo of it here in comment section so I'm here providing the link of it Original problem as I got this problem on Facebook wall of proposer. Please do check once. Thank your very much for your response. :)